Number Systems GMAS Review

Number Systems GMAS Review

7th Grade

17 Qs

quiz-placeholder

Similar activities

GCF and LCM Problems

GCF and LCM Problems

6th Grade - University

13 Qs

Motion Graphing

Motion Graphing

6th - 8th Grade

16 Qs

Interpreting Function Graphs

Interpreting Function Graphs

7th - 9th Grade

16 Qs

EOG No Calculator B

EOG No Calculator B

7th Grade

12 Qs

7th Grade Unit 1 Milestone Review

7th Grade Unit 1 Milestone Review

7th Grade

14 Qs

USC_MATH_P1

USC_MATH_P1

7th Grade

16 Qs

STAAR Integer Practice

STAAR Integer Practice

6th Grade - University

20 Qs

NO Calculator ACAP Class 3/6

NO Calculator ACAP Class 3/6

7th Grade

12 Qs

Number Systems GMAS Review

Number Systems GMAS Review

Assessment

Quiz

Mathematics

7th Grade

Hard

Created by

Breanna Handley

Used 6+ times

FREE Resource

17 questions

Show all answers

1.

MULTIPLE CHOICE QUESTION

3 mins • 1 pt

Media Image

John & Kamira are playing a game together. John's score (J) and Kamira's score (K) after round one are shown on the number line. The score John & Kamira record after round one is 2. What could this score represent?

The sum of John's score and Kamira's score

The sum of John's score and Kamira's score

The absolute value of the difference between John's score and Kamira's score

The sum of the absolute value of John's score and the absolute value of Kamira's score

Answer explanation

Media Image

The correct answer choice is (A) the sum of John's score Kamira's score. The number 2 was correctly identified as the sum of their scores. Choice (B) would give me the answer -12. Choice (C) would be the absolute value of -12, which is 12. Choice (D) is incorrect because it would give me 12.

2.

MULTIPLE CHOICE QUESTION

1 min • 1 pt

What is the value of 4.2(46 - 65)?

-798

-79.8

79.8

798

Answer explanation

Media Image

Subtracting 46 - 65 will give you a negative number. Multiplying (or distributing) the positive 4.2 by a negative will give me a negative, so A & B are the only two answers that could work. It has to be B because I am multiplying a whole number by a tenth, so my answer must be a tenth; thus the only answer that works is B!

3.

MULTIPLE CHOICE QUESTION

1 min • 1 pt

What is the value of 4.2(46 - 65)?

-59.2

-30.4

30.4

52.8

Answer explanation

Media Image

If I use PEMDAS and subtract 7 - 3.2 first, I get about 4. Multiplying -8 by 4 is -32 and the closest answer is choice (B) -30.4!

4.

MULTIPLE CHOICE QUESTION

2 mins • 1 pt

During one week, Sheila made several changes to her bank account. She made four withdrawals of $40 each from an ATM. She also used her debit card for a $156 purchase. Then she deposited her paycheck of $375. By how much did the amount in her bank account change during that week?

$59 increase

$59 decrease

$691 increase

$691 decrease

Answer explanation

Media Image

4 withdrawals means -4. -4 x 40 = -160. A $156 purchase mean -156. Deposits $375 means +375. So, -160 + (-156) + 375 = 59

5.

MULTIPLE CHOICE QUESTION

2 mins • 1 pt

Last year at a track meet, runner A and runner B both ran each of 4 laps in a race in 1.5 minutes. Runner A sets a goal for this year of changing his total time for the race by -0.12 minutes, with equal times for all 4 laps. Runner B sets a goal for this year of changing his time on each lap by -0.025 minutes. Runner A's goal is to change his time on each lap of the race by __________________ minutes.

-0.48

-0.12

-0.03

0.03

Answer explanation

Media Image

Runner A is trying to save his TOTAL time by -0.12. So to figure out how much he needs to change EACH lap by, he must DIVIDE -0.12 by 4, which will give him -0.03. Runner A must shave off .03 minutes on each lap!

6.

MULTIPLE CHOICE QUESTION

2 mins • 1 pt

Last year at a track meet, runner A and runner B both ran each of 4 laps in a race in 1.5 minutes. Runner A sets a goal for this year of changing his total time for the race by -0.12 minutes, with equal times for all 4 laps. Runner B sets a goal for this year of changing his time on each lap by -0.025 minutes. Runner B's goal is to change his total time for the race by _____________________ minutes.

-0.1

-0.025

0.025

0.1

Answer explanation

Media Image

Runner B is trying to change EACH lap by -0.025 minutes, so in order to figure out his OVERALL change, we must MULTIPLY -0.025 x 4.

7.

FILL IN THE BLANK QUESTION

2 mins • 1 pt

An employee is starting a job with an annual salary of $40,000. The employee's annual salary will increase by 3% at the end of each year with the company. What will the employee's annual salary, in dollars, be at the end of the second year with the company?

Answer explanation

Media Image

Year 0 = $40,000 salary

Year 1 = 40,000 x 1.03 = $41,200

Year 2 = 41,200 x 1.03 = $42,436

1.03 = 103% (add at the beginning since it's a raise, then you won't have to add at the end!)

Create a free account and access millions of resources

Create resources
Host any resource
Get auto-graded reports
or continue with
Microsoft
Apple
Others
By signing up, you agree to our Terms of Service & Privacy Policy
Already have an account?